Đến nội dung

Hình ảnh

$\sum_{k=1}^n\left(\lfloor k\varphi\rfloor +\left\lfloor\frac{k}{\varphi+1}\right\rfloor\right)=n^2$

* * * * * 1 Bình chọn golden-ratio floor function

  • Please log in to reply
Chủ đề này có 4 trả lời

#1
hxthanh

hxthanh

    Tín đồ $\sum$

  • Hiệp sỹ
  • 3915 Bài viết
Chứng minh rằng:
\begin{equation}\label{e1}
\sum_{k=1}^n\left(\left\lfloor k\varphi\right\rfloor +\left\lfloor\frac{k}{\varphi+1}\right\rfloor\right)=n^2
\end{equation}
Trong đó $\varphi=\frac{1+\sqrt 5}{2}$ là tỉ lệ vàng.

Bài viết đã được chỉnh sửa nội dung bởi hxthanh: 20-04-2023 - 10:31
Đánh dấu


#2
hxthanh

hxthanh

    Tín đồ $\sum$

  • Hiệp sỹ
  • 3915 Bài viết
Gợi ý: Chứng minh với $m$ nguyên dương thì
\begin{align}\label{e2}\left\lfloor m\varphi\right\rfloor-\left\lfloor\dfrac{m}{\varphi}\right\rfloor&=m\\ \label{e3}
\left\lfloor m\varphi\right\rfloor+\left\lfloor\dfrac{m}{\varphi+1}\right\rfloor&=2m-1
\end{align}

Bài viết đã được chỉnh sửa nội dung bởi hxthanh: 20-04-2023 - 11:10


#3
chanhquocnghiem

chanhquocnghiem

    Thiếu tá

  • Thành viên
  • 2494 Bài viết

Gợi ý: Chứng minh với $m$ nguyên dương thì
\begin{align*}\left\lfloor m\varphi\right\rfloor-\left\lfloor\dfrac{m}{\varphi}\right\rfloor&=m\\
\left\lfloor m\varphi\right\rfloor+\left\lfloor\dfrac{m}{\varphi+1}\right\rfloor&=2m-1
\end{align*}

$(2)\Leftrightarrow \left \lfloor m\varphi \right \rfloor-m=\left \lfloor \frac{m}{\varphi } \right \rfloor$

        $\Leftrightarrow \left \lfloor \frac{m+m\sqrt{5}}{2} \right \rfloor-m=\left \lfloor \frac{2m}{\sqrt{5}+1} \right \rfloor$

        $\Leftrightarrow \left \lfloor \frac{m\sqrt{5}-m}{2} \right \rfloor=\left \lfloor \frac{2m}{\sqrt{5}+1} \right \rfloor$

        $\Leftrightarrow \left \lfloor \frac{m(\sqrt{5}-1)}{2} \right \rfloor=\left \lfloor \frac{2m}{\sqrt{5}+1} \right \rfloor$ (cái này là hiển nhiên)


Bài viết đã được chỉnh sửa nội dung bởi chanhquocnghiem: 20-04-2023 - 13:09

...

Ðêm nay tiễn đưa

Giây phút cuối vẫn còn tay ấm tay
Mai sẽ thấm cơn lạnh khi gió lay
Và những lúc mưa gọi thương nhớ đầy ...

 

http://www.wolframal...-15)(x^2-8x+12)


#4
chanhquocnghiem

chanhquocnghiem

    Thiếu tá

  • Thành viên
  • 2494 Bài viết

Gợi ý: Chứng minh với $m$ nguyên dương thì
\begin{align}\left\lfloor m\varphi\right\rfloor+\left\lfloor\dfrac{m}{\varphi+1}\right\rfloor&=2m-1
\end{align}

Để chứng minh cái này ta sẽ sử dụng mệnh đề sau :

Mệnh đề
Nếu $a\notin \mathbb{Z}$ thì $\left \lfloor a \right \rfloor+\left \lfloor -a \right \rfloor=-1$.

(Chứng minh mệnh đề trên không có gì khó khăn)

Bây giờ ta chứng minh mệnh đề $(3)$ ở trên.

$\left \lfloor m\varphi \right \rfloor=\left \lfloor \frac{m+m\sqrt{5}}{2} \right \rfloor$

$\left \lfloor \frac{m}{\varphi+1 } \right \rfloor=\left \lfloor \frac{2m}{3+\sqrt{5}} \right \rfloor=\left \lfloor \frac{(3-\sqrt{5})m}{2} \right \rfloor=\left \lfloor \frac{3m-m\sqrt{5}}{2} \right \rfloor$

+ Nếu $m$ chẵn ($m=2k$)

   $\left \lfloor \frac{m+m\sqrt{5}}{2} \right \rfloor+\left \lfloor \frac{3m-m\sqrt{5}}{2} \right \rfloor=k+3k+\left \lfloor \frac{m\sqrt{5}}{2} \right \rfloor+\left \lfloor \frac{-m\sqrt{5}}{2} \right \rfloor=4k-1=2m-1$

+ Nếu $m$ lẻ ($m=2k+1$)

   $\left \lfloor \frac{m+m\sqrt{5}}{2} \right \rfloor+\left \lfloor \frac{3m-m\sqrt{5}}{2} \right \rfloor=\left \lfloor \frac{2k+1+(2k+1)\sqrt{5}}{2} \right \rfloor+\left \lfloor \frac{6k+4-1-(2k+1)\sqrt{5}}{2} \right \rfloor$

$=k+3k+2+\left \lfloor \frac{1+(2k+1)\sqrt{5}}{2} \right \rfloor+\left \lfloor \frac{-1-(2k+1)\sqrt{5}}{2} \right \rfloor=4k+2-1=2m-1$
 


Bài viết đã được chỉnh sửa nội dung bởi chanhquocnghiem: 20-04-2023 - 14:32

...

Ðêm nay tiễn đưa

Giây phút cuối vẫn còn tay ấm tay
Mai sẽ thấm cơn lạnh khi gió lay
Và những lúc mưa gọi thương nhớ đầy ...

 

http://www.wolframal...-15)(x^2-8x+12)


#5
chanhquocnghiem

chanhquocnghiem

    Thiếu tá

  • Thành viên
  • 2494 Bài viết

Chứng minh rằng:
\begin{equation}\label{e1}
\sum_{k=1}^n\left(\left\lfloor k\varphi\right\rfloor +\left\lfloor\frac{k}{\varphi+1}\right\rfloor\right)=n^2
\end{equation}
Trong đó $\varphi=\frac{1+\sqrt 5}{2}$ là tỉ lệ vàng.

Áp dụng kết quả trên, ta có :

$\sum_{k=1}^{n}\left ( \left \lfloor k\varphi \right \rfloor+\left \lfloor \frac{k}{\varphi +1} \right \rfloor \right )=\sum_{k=1}^{n}(2k-1)=2.\frac{n(n+1)}{2}-n=n^2$.
 


...

Ðêm nay tiễn đưa

Giây phút cuối vẫn còn tay ấm tay
Mai sẽ thấm cơn lạnh khi gió lay
Và những lúc mưa gọi thương nhớ đầy ...

 

http://www.wolframal...-15)(x^2-8x+12)






Được gắn nhãn với một hoặc nhiều trong số những từ khóa sau: golden-ratio, floor function

1 người đang xem chủ đề

0 thành viên, 1 khách, 0 thành viên ẩn danh